Trayectoria del proyectil lanzado cuesta abajo

Estoy aprendiendo mecánica y me dispuse a resolver un problema para determinar el ángulo óptimo para lanzar un proyectil cuando estoy parado en una colina, para lograr el máximo alcance. Mi respuesta parece casi plausible, excepto por un término que, para ser plausible, debe cambiar de signo. Pero no puedo encontrar ningún agujero en mi razonamiento.

Problema: Estoy parado en una colina recta con pendiente descendente y deseo lanzar una piedra para alcanzar el máximo alcance. La colina está inclinada hacia abajo desde la horizontal por φ . que angulo θ por encima de la horizontal debo tirarlo?

Mi solución:

  1. Usa coordenadas para que X es paralelo a la colina

  2. Dejar α = φ + θ (es decir, el ángulo sobre el suelo al que estoy lanzando)

  3. Entonces inicial v es v X = porque α , v y = pecado α (normalizando las unidades para eliminar cualquier constante)

  4. Entonces la aceleración de la gravedad es a X = k pecado φ , a y = k porque φ (la gravedad está en la dirección y). Para simplificar los cálculos, suponga k = 2 (la respuesta es válida para cualquier valor de la gravedad, por lo que es igual en la Luna que en la Tierra)

  5. Queremos encontrar el alfa que maximiza s X en el momento que hace s y = 0 . Primero, encuentra el tiempo que hace s y = 0 ; llámalo t.

  6. s y = t pecado α t 2 porque φ . Usando la fórmula cuadrática, s y = 0 en t = 0 o t = pecado α porque φ .

  7. Ahora, encuentra s X en esto t . Sustituyendo y usando álgebra básica y trigonometría, obtenemos s X = pecado α porque α pecado 2 α broncearse φ . (Esto tiene sentido; el primer término tiene un máximo en π / 4 , como esperaríamos de la simetría. El segundo término nos dice que si el suelo se inclina significativamente, debemos reducir nuestro ángulo de lanzamiento. Muy plausible.)

  8. Tomando phi como una constante, deseamos maximizar esta expresión. Un poco de cálculo e identidades trigonométricas hace que la derivada sea igual a porque ( 2 α ) pecado ( 2 α ) broncearse φ , que tiene un cero en α = π / 4 φ / 2 , o θ = π / 4 3 φ / 2 . Aquí es donde las cosas se rompen. El primer término, π / 4 , parece correcto. Pero el segundo término da resultados ridículos.

  9. Cambiar el signo del segundo término en la ecuación alfa termina con θ = π / 4 φ / 2 , lo que da resultados completamente plausibles. ¡Pero no puedo encontrar ningún error en mi razonamiento o cálculos!

¿Alguien puede encontrar el eslabón perdido?


Explicación de la respuesta:

Como determinó Pygmalion, el paso 4 es incorrecto. El a y el valor es correcto, pero , a X debe ser positivo : apuntando hacia abajo de la colina.

La respuesta es independiente de la magnitud de la gravedad; pero depende de la dirección .

Revisando la derivación:

7. s X = pecado α porque α + pecado 2 α broncearse ϕ

8. La derivada es porque ( 2 α ) + pecado ( 2 α ) broncearse ϕ , con cero en α = π / 4 + ϕ / 2 , de este modo θ = π / 4 ϕ / 2 . QED.

Bienvenido a PhysSE. Por favor, la próxima vez use el formato matemático para que x = sin theta se convierta en X = pecado θ Por ejemplo.
Un punto: afirmo que la respuesta es independiente de la gravedad, lo mismo en la Luna y Marte. Pero, ¿qué pasa con la "gravedad hacia atrás" (alejándose del suelo)? Eso seguramente cambiaría la respuesta. Esto no resuelve las cosas, pero puede ser un buen comienzo.
¿Por qué no mantener el sistema de coordenadas verticales horizontales (por X , y componentes) y hacer coincidir la trayectoria y ( t ) , X ( t ) a la pendiente descendente de broncearse φ o y ( t ) = X ( t ) broncearse φ ?
@S.RobertJames: creo que la respuesta depende de la relación entre la velocidad inicial y la gravedad.
Bien, es independiente de gramo . mi fórmula es R = 2 tu 2 gramo porque φ ( porque θ pecado ( φ + θ ) ) , máximo en θ = π 2 φ 2 . ¿Lo que es tuyo?
@Manishearth Máximo debe ser π / 4 + algo Verifica el límite φ = 0
@Pygmalion: Sí, lo es π 4 (Lo copié mal de mi bloc de notas), pero sigo teniendo un signo negativo. ¿ Estás seguro de que estás tirando la pelota hacia abajo ? En ese caso, hay una ventaja.
@Manishearth Tal vez sea así. Calcular a la baja parece un problema más intuitivo que al contrario. S.RobertJames nos dirá...

Respuestas (1)

Por lo que yo veo, a y = k porque ( φ ) < 0 , pero a X = k pecado ( φ ) > 0 ! Al menos si estás lanzando el proyectil en dirección hacia abajo...

yo creo que son los dos < 0 . a se debe a la gravedad, y la gravedad es recta hacia abajo, girada en el sentido de las agujas del reloj por ϕ debido a la rotación de coordenadas. Giratorio [ 0 1 ] por ϕ , 0 < ϕ < π / 2 produce ambos componentes negativos.
@S.RobertJames Si está lanzando hacia abajo , la distancia máxima es mayor , esto se debe a que a y es mas pequeño y a X es positivo (te ayuda). Olvídese de rotar, dibuje la imagen.
Por favor, explique por qué se siente a X es positivo (ayuda). Aunque veo su punto de manera intuitiva (la gravedad me ayuda a arrojarme cuesta abajo), la rotación de las coordenadas (como se describe en mi comentario anterior) no produce un resultado negativo (dolor) a X ?
yo leo el 0 ! como "factorial cero". Me hizo decir "espera, ¿qué?". Luego volví a mirar la pregunta para asegurarme de que no sea recursiva (donde hay múltiples rebotes). ;-)
Entendido: los ejes de coordenadas giran en el sentido de las agujas del reloj. Los vectores de las coordenadas iniciales giran en sentido antihorario. Por eso, a X gira para ser positivo! Excelente, gracias Pygmalion. tu numero mi \i π + 1 ! (quitando mi sombrero a Manishearth).
Sí, S.Robert James, a veces es una buena idea esbozar el problema. :) Si esto responde a su pregunta, es posible que desee aceptarla haciendo clic en el botón verde debajo de la pregunta :)
S. Robert James: Encantado de verte. :)
@Manishearth ¡No entendí bien 0! comentario. ¿Estaba dirigido hacia mí?
@Pygmalion también lo usé, en mi galardón :-)
Ups, escribí infierno en lugar de ayuda. Pero supongo que recibiste el mensaje correcto. :)